You are on page 1of 31

Series

Objectives: At the end of the module students are able to

⋄ Identify the difference between sequence and series

⋄ Identify the properties of some special series such as geometric series, harmonic
series and p–series

⋄ Determine whether a series converges or diverges using test of convergence

⋄ Evaluate the radius and interval of convergence of a series

§1 Infinite Series
Infinite series is formed by adding terms of an infinite sequence:

Sequence: {an }∞
n=1 = a1 , a2 , a3 , · · · , an , · · ·
P∞
Series: n=1 an = a1 + a2 + a3 + · · · + an + · · ·

Definition 1.1:

The kth partial sum Sk of the series is the sum of first k terms:
k
X
Sk = an = a1 + a2 + · · · + ak
n=1

For example, the partial sum of first few terms are

S1 = a1
S2 = a1 + a2
S3 = a1 + a2 + a3
S100 = sum of first 100 terms of an

Example:
 
1 1 1 1
(i) Sequence: n
= , , ,···
2 2 4 8

X 1 1 1 1
Series: n
= + + +···
n=1
2 2 4 8

1
 
(−1)k+1 1 1 1
(ii) Sequence: = −1 , , − , , · · ·
k+1 2 3 4
∞ k+1
X (−1) 1 1 1
Series: = −1 + + − + +···
k=0
k + 1 2 3 4

Definition 1.2:

th
Given a sequence {ak }∞ k=1 = {a1 , a2 , a3 , · · · }, the n partial sum
P Sn is the sum of
first n terms of the sequence, i.e., Sn = a1 + a2 + a3 + · · · + an = nk=1 ak . The partial
sum of first n terms are

S1 = a1
S2 = a1 + a2
S3 = a1 + a2 + a3
..
.
Sn = a1 + a2 + a3 + · · · + an

The column on the left (in red) forms a sequence

{Sk }nk=1 = S1 , S2 , S3 , · · · , Sn

If the sequence of the partial sums {Sn } converges to L, then the sum of the series
converges to L, i.e.,

lim Sn = L
n→∞

X
S∞ = an = L
n=1
P
where L is a finite number. The limit L is the sum of the infinite series an and we
may write it as

X
L= an = a1 + a2 + a3 + a4 + · · ·
n=1

If the sequence of partial sum {Sn } is not convergent, then it is said to be divergent.

Example 1.1:
Find the limit of the infinite series ∞
X 1
n=1
2n

2
Solution:
We write down the first four partial sum of the series
1
S1 =
2
1 1 3
S2 = + =
2 4 4
1 1 1 7
S3 = + + =
2 4 8 8
1 1 1 1 15
S4 = + + + =
2 4 8 16 16
and from the pattern of the values we conclude that the partial sum is
2n − 1 1
Sn = n
=1− n
2 2
So that the limit  1
lim Sn = lim 1− n = 1
n→∞ n→∞ 2
The infinite sum of the series ∞
X 1
S∞ = =1
n=1
2n
converges to 1. 

Example 1.2:
Find the limit of the infinite series

X 1
n=1
n(n + 1)

Solution:
We write down the first four partial sum of the series
1
S1 =
2
1 1 2
S2 = + =
2 6 3
1 1 1 3
S3 = + + =
2 6 12 4
1 1 1 1 4
S4 = + + + =
2 6 12 20 5
and from the pattern of the values we conclude that the partial sum is
n
Sn =
n+1

3
The infinite sum of the series
n 1
S∞ = lim Sn = lim = lim 1 =1
n→∞ n→∞ n + 1 n→∞ 1 +
n

converges to 1. 

Example 1.3:
Determine whether the series with the sequence

1 , −1 , 1 , −1 , 1 , −1 , · · ·

is convergent/divergent.

Solution:
1 1
S1 = 1 = +
2 2
1 1
S2 = 1 + (−1) = 0 = −
2 2
1 1
S3 = 1 + (−1) + 1 = 1 = +
2 2
1 1
S4 = 1 + (−1) + 1 + (−1) = 0 = −
2 2
and from the pattern of the values we conclude that the partial sum is
1 1
Sn = + (−1)n+1
2 2
We may write (
1 if n is odd
Sn =
0 if n is even
Since {Sn } oscillates between 1 and 0, it follows that limn→∞ Sn does not exist, hence
the series diverges. 

§2 Geometric Series
Geometric Series is a series of numbers having a common ratio r such that
n
X
Sn = ar k−1 = a + ar + ar 2 + ar 3 + · · · + ar n−2 + ar n−1
k=1

4
where a is the first term and r is the ratio. We note that the nth term is an = ar n−1 .
As we multiply the series with r, we have
n
X
r · Sn = ar k = ar + ar 2 + ar 3 + ar 4 + · · · + ar n−1 + ar n
k=1

Then we subtract the first series with the second series and find that
r · Sn − Sn = ar n − a
(r − 1)Sn = a(r n − 1)
a(r n − 1)
∴ Sn =
r−1
Then, the infinite sum of the geometric series is
 a(r n − 1) 
lim Sn = lim
n→∞ n→∞ r−1
 ar n a 
= lim −
n→∞ r − 1 r−1
If the ratio |r| < 1, the first term on the right hand side vanishes (please refer to Theorem
3 in Sequence),
a
∴ S∞ =
1−r
So that the geometric series converges if |r| < 1 and diverges if |r| > 1.

Example 2.1:
Determine whether the series converges/diverges,
10 20 40
5− + − +···
3 9 27
Solution:
The series is also equivalent to
 2  2 2  2 3
5 + (5) − + (5) − + (5) − +···
3 3 3
Therefore a = 5 and r = − 23 . Since |r| = 23 < 1, then the series is convergent and the
sum of the infinite series is
a 5
S= = =3
1−r 1 − (− 23 )

Example 2.2:
Determine whether the series conveges/diverges,
1 + 0.4 + 0.16 + 0.064 + · · ·

5
Solution:
The series can be rewritten as
1 + 0.4 + (0.4)2 + (0.4)3 + · · ·
Therefore, a = 1 and r = 0.4. Since |r| < 1, the series is convergent and the sum of the
infinite series is
1 5
S= = = 1.66
1 − 0.4 3

Example 2.3:
Write the number
2.317 = 2.31717171717...
as a fraction (ratio of integer).
Solution:
The number can be rewritten as
2.31717171717...
= 2.3 + 0.017 + 0.00017 + 0.0000017 + · · ·
17 17 17
= 2.3 + + + +···
1000 100000 100000
17 17 17
= 2.3 + 3
+ 5
+ +···
(10) (10) (10)7
17 17 1 17 1
= 2.3 + 3
+ 3
· 2
+ · +···
(10) (10) (10) (10) (10)4
3

↑ ↑ ↑ ↑ ↑
a + a × r + a × r2 + · · ·
17
From the second term onward are of the form of Geometric Series with a = and
(10)3
1
r= . Thus,
(10)2
a
∴ 2.317 = 2.3 +
1−r
17
(10)3
= 2.3 + 1
1 − (10) 2

23 17 1147
= + =
10 990 495

Example 2.4:
Is the series ∞
X
22n 31−n
n=1

6
convergent/divergent?

Solution:

X ∞
X
22n 31−n = (22 )n · 3 · 3−n
n=1 n=1

X 1
= 4n · 3 ·
n=1
3n
∞  n
X 4
= 3 n
n=1
3
X 4 n
∞ 
= 3
n=1
3

4
The infinite sum is a Geometric Series with a = 3
and r = 43 . Since |r| > 1, the series
diverges. 

Exercise:

1.) Show that the series


2 2 2
2+ + 2 +···+ n +···
3 3 3
converges and find its sum.

2.) Determine whether the series converges, if so, find the infinite sum.
1 1 1
i) − + −1+···
8 4 2

X (−3)n−1
ii)
n=1
4n

§3 Telescoping Series
Telescoping Series is a series whose sum appears to have the situation that almost every
term cancels with either a preceding or succeeding term. For example, the series
∞ ∞  
X 1 X 1 1
S∞ = = −
n=1
n(n + 1) n=1
n n+1

7
we consider the partial sums of this series up to kth term
k  
X 1 1
Sk = −
n=1
n n+1
       
1 1 1 1 1 1
= 1− + − + − + −···
2 2 3 3 4 4
     
1 1 1 1 1
+···+ ···− + − + −
k−1 k−1 k k k+1

We can see that the terms with the same colour are canceled with each other. Thus, the
terms which survive are the first and the last number:
1
Sk = 1 −
k+1
Therefore, the sum of the infinite series is
 
1
S∞ = lim Sk = lim 1 − =1
k→∞ k→∞ k+1

∴ The series converges to 1.

§4 Harmonic Series
The harmonic series, given by

X 1 1 1 1
= 1+ + + +···
n=1
n 2 3 4

is divergent infinite series in spite of the fact that the limit of the sequence when n goes
to infinity is zero. The proof of the divergence of the harmonic series is shown below by
comparing the harmonic series with another divergent series:
1 1 1 1 1 1 1
1+
+ + + + + + +···
2 3 4 5 6 7 8
1 1 1 1 1 1 1
> 1+ + + + + + + +···
2 |4 {z 4} |8 8 {z 8 8}
1 1
= =
2 2
where each colour term of the harmonic series is greater than the corresponding term
in the second series with the same colour, and therefore the sum of the harmonic series

8
must be greater than the sum of the second series. However, the sum of the second
series is infinite,
     
1 1 1 1 1 1 1
1+ + + + + + + +···
2 4 4 8 8 8 8
     
1 1 1
= 1+ + + +··· = ∞
2 2 2

It follows that the infinite sum of the harmonic series must be divergent.

Example 4.1:
Show that each of the following series are divergent.
∞ ∞
X 3 X 1
(i) (ii)
n=1
n n=3
n

Solution:

(i) By the fact that constant can be factored out of the series as follows,
∞ ∞
X 3 X 1
=3
n=1
n n=1
n
P 1
Since ∞ n=1 n is divergent, any constant multiplies a divergent series it will still be
divergent and so the series has to be divergent.

(ii) The first two terms of the harmonic series is


∞ ∞
X 1 1 X1
= 1+ +
n=1
n 2 n=3 n

3 X1
= +
2 n=3 n
∞ ∞
X 1 X 1 3
=⇒ = −
n=3
n n=1
n 2

As we subtract a constant from a divergent series, the series will still be divergent.
In order word, an infinity minus a constant which is still infinity. So that the series
is divergent.

9
§5 Determining Convergence/Divergence Of Series

5.1 n–th Term Test



X
⋄ If lim an 6= 0 then the series an is divergent.
n→∞
n=1

⋄ If lim an = 0, further investigation is necessary to determine whether the series


n→∞
X∞
an is convergent or divergent. In other word, it is not guarantee that the series
n=1
converges.

Example 5.1:
Determine whether the series ∞
X n2
n=1
5n2 + 4
is convergent/divergent.

Solution:
By using n–th term test,

n2 1 1
lim 2
= lim 4 = 6= 0
n→∞ 5n + 4 n→∞ 5 + 2 5
n

thus the series diverges. 

Example 5.2:
Determine the following series, by using n–th term test, whether they are conver-
gent/divergent?

1. Given the series: ∞


X n
n=1
2n + 1
By using n–th term test:
n 1
lim = 6= 0
n→∞ 2n + 1 2
Conclusion: Diverges

2. Given the series: ∞


X 1
n=1
n2

10
By using n–th term test:
1
lim=0
n→∞ n2

Conclusion: Further investigation is needed.

3. Given the series: ∞


X 1

n=1
n
By using n–th term test:
1
lim √ = 0
n→∞ n
Conclusion: Further investigation is needed.

4. Given the series: ∞


X en
n=1
n
By using n–th term test:
en en
lim = lim =∞
n→∞ n n→∞ 1

Conclusion: Diverges 

Remark:
If limn→∞ an = 0, then the series may either converges or diverges. We need to use other
test techniques to confirm.

Theorem 1:

Given a series ∞
X
an = a1 + a2 + a3 + · · ·
n=1

For any positive integer k, if the sum of the same series is taken from (k + 1)–th term
onwards ∞
X
an = ak+1 + ak+2 + ak+3 + · · ·
n=k+1

then both series are either convergent or divergent.

Example 5.3:
Show whether the following series

X 1 1 1 1
= + + +···
n=1
(n + 2)(n + 3) 3·4 4·5 5·6

11
convergent/divergent.

Solution:
Recall the series we have considered earlier (Telescoping series):

X 1 1 1 1 1 1
= + + + + +···
n=1
n(n + 1) 1·2 2·3 3·4 4·5 5·6

1 1 X 1
= + +
1 · 2 2 · 3 n=1 (n + 2)(n + 3)

The series in the question is the sum of this series started from third term onwards, this
means that ∞ ∞
X 1 X 1
=
n=1
(n + 2)(n + 3) n=3 n(n + 1)
P 1
As we found before the series ∞ n=1 n(n+1) converges to 1, and thus by the above Theorem,
the series in the question is convergent and converges to

X 1 1 1 1
S∞ = =1− − =
n=1
(n + 2)(n + 3) 1·2 2·3 3


Theorem 2:
P P
If an and bn are convergent series with sums A and B, respectively, then
P
i) (an + bn ) converges with sum A + B
P
ii) (c × an ) converges with sum c × A, for every c 6= 0
P
iii) (an − bn ) converges with sum A − B

Example 5.4:
Find the sum of series ∞  
X 3 1
+ n
n=1
n(n + 1) 2

Solution:
By using Theorem 2,
∞   ∞ ∞
X 3 1 X 1 X 1
+ n =3 +
n=1
n(n + 1) 2 n=1
n(n + 1) n=1 2n

From the previous results where the first series is a telescoping series and the second
series has been considered in Example 1.1, the sum of the series given in the question is

12
convergent and converges to 3 × 1 + 1 = 4 

Example 5.5:
Find the sum of series ∞  
X 1 1
+
n=1
5n n

Solution:
By using the theorem,
∞   X ∞ ∞
X 1 1 1 X 1
+ = +
n=1
5n n n=1
5n n=1 n
Since the first series is a geometric series with r = 15 < 1, this series is convergent. The
second series is a harmonic series, it is divergent. The sum of these two series is thus
divergent. 

5.2 p–Series
The p–series is a series of the following form

X 1 1 1 1 1
p
= p
+ p
+ p
+ p
+···
n=1
n 1 2 3 4

Theorem 3:

Given a p–series,

X 1
n=1
np

⋄ If p > 1, the series is convergent.


⋄ If p ≤ 1, the series is divergent.

e.g.,
The series ∞
X 1 1 1 1 1
4
= 4 + 4 + 4 + 4 +···
n=1
n 1 2 3 4
is convergent because it is a p–series with p = 4 > 1.

The series ∞
X 1 1 1 1 1
1/2
= 1/2 + 1/2 + 1/2 + 1/2 + · · ·
n=1
n 1 2 3 4
is divergent because it is a p–series with p = 1/2 < 1.

13
Exercise:
Determine whether the following series is convergent or divergent:
1 1 1 1
i) 1 + + + + +···
8 27 64 125

X
ii) n−1.4 + 3n−1.2
n=1

X (n + 1)2
iii)
n=1
n(n + 2)

5.3 Comparison Test


Assume 0 ≤ an ≤ bn for all n.
P P
⋄ Suppose that the sum of series bn is convergent, then the series an is also
convergent. In other word, a series of positive terms is convergent if its terms
are less than the corresponding terms of a positive series which is known to be
convergent.
P P
⋄ Suppose that the sum of series an is divergent, then the series bn is also
divergent. In other word, a series is divergent if its terms are greater than the
corresponding terms of a series which is known to be divergent.

Example 5.6:
Determine whether the series ∞
X 5
n=1
2n2 + 4n + 3
converges/diverges.

Solution:
Since n ≥ 1, thus 2n2 + 4n + 3 > 2n2 , then
5 5
<
2n2 + 4n + 3 2n2
The series ∞ ∞
X 5 5X 1
=
n=1
2n2 2 n=1 n2
is a p–series with p = 2 > 1, by Theorem 3 the series is convergent. Therefore, by
Comparison Test the series

X 5
is convergent .
n=1
2n2 + 4n + 3

14

Example 5.7:
Determine whether ∞
X ln n
n=3
n
converges/diverges.

Solution:
Since ln(e) = ln(2.7183) = 1, therefore, ln n > 1 for n ≥ 3. Thus,
ln n 1
> for n ≥ 3 .
n n
P 1
We know that
P∞ the series ∞ n=1 n is a harmonic series and it is divergent. By Theorem 1
1
the series n=3 n is also divergent, and by Comparison Test,

X ln n
diverges .
n=3
n

Exercise:
Determine, by using Comparison Test, whether the series converges or diverges:
∞ ∞
X 2 X 4 + 3n
i) ii)
n=1
n4 + 2 n=1
2n

5.4 Limit Comparison Test


Given the positive terms series

X ∞
X
an and bn
n=1 n=1

We require that all an and bn are positive. Suppose


an
lim =c
n→∞ bn

where c > 0 is a finite number.


an
⋄ If the limit of bn
is positive,
an
lim = c > 0,
n→∞ bn
P P∞
then the sums ∞ n=1 an and n=1 bn are either both convergent or both divergent.
In other word, if the limit is positive, the terms of these two series are growing at
the same rate, so either both series converge or diverge together.

15
P P
⋄ If the limit of abnn is zero, and the sum bn converges, then the sum an converges.
In other word, if the limit is zero, the bottom terms are growing more quickly than
the upper terms. So that if the bottom series converges, the upper series which is
growing more slowly must also converge.
P P
⋄ If the limit of abnn is infinite, and the sum bn diverges, then the sum an diverges.
In other word, if the limit is infinite, the bottom series is growing more slowly. If
the bottom series diverges, the upper series must also diverge.

Remark:
The Limit Comparison Test only applies to series with non-negative terms, i.e., an , bn >
0 for all n.

Example 5.8:
By using Limit Comparison Test, determine whether the series

X 1
n=1
2n −1

converges/diverges.

Solution:
Let
1
an =
2n −1
We choose
1
bn =
2n
Then,
1
an 2n −1 1
lim = 1 = lim =1>0
n→∞ bn n→∞ 1 − 1n
2n 2

X 1
Since the series is a convergent geometric series (please see Example 1.1), by
n=1
2n

X 1
Limit Comparison Test, the series n
converges. 
n=1
2 −1

Example 5.9:
By using Limit Comparison Test, determine whether the series

X 2k 2 + 3k

k=1
5 + k5

16
converges/diverges.

Solution:
The highest power of the numerator is 2k 2 and the highest power of the denominator
proportional to k 5/2 . We let
2k 2 + 3k
ak = √
5 + k5
and choose
2k 2 2
bk = √ = 1/2
k5 k
Then the limit
2k 2
√ +3k
an 5+k 5
lim = lim 2
k→∞ bn k→∞
k 1/2
2
2k + 3k k 1/2
= lim √ ×
k→∞ 5 + k5 2
2 3
k (2 + k ) k 1/2
= lim q ×
k→∞ 5/2
k 5
+1 2
k5

2 + k3
= lim q =1>0
k→∞
2 k55 + 1
P∞ 1
Since n=1 bk is p–series with p = 2
, it is divergent. By Limit Comparison Test,

X 2k 2 + 3k
√ diverges. 
k=1
5 + k5

Exercise:
By Limit Comparison Test, determine whether the following series converges or diverges.
∞ ∞
X 4k 2 − 2k + 6 X 5
i) ii)
k=1
8k 7 +k−8 n=1
3n +1

17
5.5 The Ratio Test

X
Given a series an .
n=1


an+1 X
i) If lim
= L < 1, then the series an is convergent.
n→∞ an n=1

an+1 X
ii) If lim
= L > 1, then the series an is divergent.
n→∞ an n=1

an+1 X
iii) If lim
= L = 1, then the series an may be convergent or convergent.
n→∞ an n=1

Example 5.10:
Use the ratio test, determine whether the given series is convergent/divergent:
∞ ∞ ∞
X 1 X (2k)! X 1
i) ii) iii)
n=1
n! k=1
4k k=1
2k − 1

Solution:
1
i) Let an = n!
. By ratio test,
1
an+1
lim = lim (n+1)!
1
n=∞ an n=∞
n!


n!
= lim
n=∞ (n + 1)!

1
= lim =0<1
n=∞ n + 1

Therefore, the series is convergent.


(2k)!
ii) Let ak = 4k
. By ratio test,

2(k+1) !
ak+1
lim = lim 4k+1
k=∞ ak k=∞ (2k)!
4k

(2k + 2)! 1
= lim ×
k=∞ (2k)! 4

1 (2k)!(2k + 1)(2k + 2)
= lim
4 k=∞ (2k)!
1
= lim (2k + 1)(2k + 2) = +∞
4 k=∞

18
Therefore, the series is divergent.
1
iii) Let ak = 2k−1
. By ratio test,
1

ak+1
lim = lim 2(k+1)−1
1
k=∞ ak k=∞
2k−1


2k − 1
= lim
k=∞ 2k + 1

2 − k1
= lim =1
k=∞ 2 + 1
k

Therefore, the series may be converges or diverges. 

Exercise:
Use ratio test to determine whether the following series are convergent/divergent.
∞ ∞
X 3k X 1
i) ii)
k=1
k! n=1
5n

5.6 The Root Test



X
Consider the sum an .
n=1


X
1
1. If lim |an | = L < 1, then the series
n an converges absolutely.
n→∞
n=1


X
1
2. If lim |an | = L > 1, then the series
n an diverges.
n→∞
n=1
1
3. If lim |an | n = 1, then we may conclude nothing from this (inconclusive).
n→∞

Example 5.11:
Use the Root Test to determine whether the following series
∞  k ∞
X 4k − 5 X 1
i) ii) n
k=1
2k + 1 n=1
ln(n + 1)

converge or diverge.

19
Solution:

i) By using Root Test,


 1
4k − 5 k k 4k − 5
lim = lim
k→∞ 2k + 1 k→∞ 2k + 1

4 − k5
= lim =2>1
k→∞ 2 + 1
k

Therefore, the series diverges.

ii) By using Root Test,


  n1  n  n1
1 1
lim n = lim
n→∞ ln(n + 1) n→∞ ln(n + 1)
1
= lim =0<1
n→∞ ln(n + 1)

Therefore, the series converges.

Exercise:
Determine, by using Root Test, whether the following series
∞  n ∞
X 3n + 1 X nn
1) 3)
n=1
2n − 1 k=1
2 × (16n )
∞ 
X k ∞
X 1
−k
2) 1−e 4) nn
k=1 k=1

converges/diverges.

5.7 The Alternating Series Test (AST)


An alternating series is a series whose terms are alternately positive and negative, e.g.,

X (−1)n−1 1 1 1 1 1
=1− + − + − +···
n=1
n 2 3 4 5 6

20
The alternating series test is used when the terms of the underlying sequence alternate.
Suppose that we have a series

X
(−1)n−1 an = a1 − a2 + a3 − a4 + · · ·
n=1

where an ≥ 0 for all n ∈ N. Then if

i) {an } is a decreasing sequence, i.e., a1 ≥ a2 ≥ a3 ≥ · · · , and


ii) lim an = 0,
n→∞

then the series is convergent.

Example 5.12:
Use AST determine whether the following series
∞ ∞
X (−1)n−1 X (−1)k+1 (k + 3)
1) 2)
n=1
n k=1
k(k + 1)

converges/diverges.

Solution:

1) The series is given by



X (−1)n−1 1 1 1 1 1
=1− + − + − +···
n=1
n 2 3 4 5 6
1 1
i) we observe that an > an+1 because n
> n+1
, and
1
ii) limn→∞ an = limn→∞ n
= 0.
Thus, both conditions of AST are satisfied and it may conclude that the series is
convergent.
2) i) To show ak > ak+1 ,
(k + 3) k+4
ak − ak+1 = −
k(k + 1) (k + 1)(k + 2)
(k + 2)(k + 3) − k(k + 4)
=
k(k + 1)(k + 2)
k + 5k + 6 − k 2 − 4k
2
=
k(k + 1)(k + 2)
k+6
= >0
k(k + 1)(k + 2)

21
for all k ≥ 1. We found that ak − ak+1 > 0 ⇒ ak > ak+1 which satisfies the first
condition of AST.
ii) The limit
k+3 1 + k3
lim = lim =0
k→∞ k(k + 1) k→∞ k + 1

Thus, the second condition of AST is satisfied and the series converges.

Exercise:
Use AST determine whether the following series
∞ ∞
X (−1)n+1 X
1) 2) (−1)k+1 e−k
n=1
n2 k=1

converges/diverges.

§6 Types of Convergence
P P
A series an is called absolutely convergent if the series of absolute value of |an | is
convergent. E.g., the series

X (−1)n−1 1 1
=1− + 2 −···
n=1
n2 2 2 3

is absolutely convergent because


∞ ∞
X (−1)n−1 X 1 1 1
= = 1 + + +···
n2 n2 22 32
n=1 n=1
P P
is convergent because it is a p–series with p = 2 > 1. If an is convergent and |an |
is divergent, we call the series conditionally convergent.

Theorem P 4:
If a series an is called absolutely convergent, then the series is convergent.

Example 6.1:
The series ∞
X (−1)n−1 1 1
=1− + −···
n=1
n 2 3

22
is convergent (please see Example 5.12), however the series
∞ ∞
X (−1)n−1 X 1 1 1
= = 1+ + +···
n=1
n
n=1
n 2 3
is divergent (because it is Harmonic series). Therefore, the series is conditionally con-
vergent. 

6.1 The Ratio Test for Absolute Convergence


P
Given an alternating series ∞ n=1 an .

an+1 P∞
⋄ If lim = L < 1, then the series
n=1 an is absolutely convergent (and
n→∞ an
therefore converges).

an+1 P
⋄ If lim = L > 1, then the series ∞ an diverges.
n=1
n→∞ an

an+1
⋄ If lim = 1, then the series P∞ an may either be convergent or divergent
n=1
n→∞ an
(inconclusive).

Example 6.2:
Determine the type of convergence of the following series

X n3
(−1)n
n=1
3n

Solution:
Let
n3 (n + 1)3
an = (−1)n , then an+1 = (−1)n+1
3n 3n+1
By using ratio test

an+1 (−1)n+1 (n + 1)3 /3n+1
lim = lim
n→∞ an n→∞ (−1)n n3 /3n
(n + 1)3 3n
= lim × 3
n→∞ 3n+2 n
 3
1 n+1
= lim
n→∞ 3 n
 3
1 1 + n1 1
= lim =
n→∞ 3 1 3
Therefore, by ratio test, the series converges absolutely. 

23
§7 Power Series
Objective: At the end of the module, you should be able to

⋄ Identify properties of the power series,

⋄ determine radius of convergence,

⋄ determine interval of convergence, represent some functions using power series.

7.1 What is Power Series?


It is a series of the form

X
cn xn = c0 + c1 x + c2 x2 + c3 x3 + · · ·
n=0

where x is variable and cn ’s are constant coefficient of the series.


E.g.,

X
i) xn = 1 + x + x2 + x3 + · · ·
n=0


X (−1)n x2n+1 x3 x5
ii) =x− + −···
n=0
2n + 1 3 5

For each fixed x value, the series becomes as what we had learned earlier (no variable) and
we can use the appropriate techniques to test whether the series converges or diverges.
E.g.,
X∞
xn = 1 + x + x2 + x3 + · · ·
n=0

If x = 2, the series becomes



X
2n = 1 + 2 + 22 + 23 + · · ·
n=0

which is divergent.
1
If x = , the series becomes
2
∞  n  2  3
X 1 1 1 1
=1+ + + +···
n=0
2 2 2 2

which is convergent (geometric series with r = 1/2).

24
What we need to do here is to find what is/are the possible value of x that will make
the series converges. There is a number R such that the power series will converge for
|x| < R and will diverge for |x| > R. The number R is called the radius of convergence
of the series. This can be written in interval form as
|x| < R ⇒ −R < x < R
This is called interval of convergence. Note that the series may or may not converge if
|x| = R. There are four possibilities of interval of convergence:

i) −R < x < R , (−R, R)


ii) −R ≤ x < R , [−R, R)
iii) −R < x ≤ R , (−R, R]
iv) −R ≤ x ≤ R , [−R, R]

Therefore, we must check whether the end point values should be included or not so
that the power series will converge for these values. To completely identify the interval
of convergence all that we have to do is determine if the power series will converge for
x = −R and x = R.
E.g.,
X∞
xn = 1 + x + x2 + x3 + · · ·
n=0
This is a geometric series, we know for this series to converge when
|x| < 1 or −1 <x <1
Here, the radius of convergence R = 1. The end point values −1 and 1 are not inclusive.

Example 7.1:
For what values of x is the series ∞
X
n!xn
n=0
converges?

Solution:
By Ratio Test,

an+1 (n + 1)!xn+1
lim = lim
n→∞ an n→∞ n!xn

n!(n + 1)xn · x
= lim
n→∞ n!xn
= lim |(n + 1)x|
n→∞
= lim (n + 1)|x| = ∞
n→∞

25
The series diverges for all values of x except when x = 0, thus, the radius of convergence
R = 0. 

Example 7.2:
Find the radius of convergence and the interval of convergence of the series

X (−1)n xn
n=0
n+1

Solution:
By Ratio Test,
n+1
(−1)n+1 xn+1 /(n + 2) x n + 1
lim = lim ×
n→∞ (−1)n xn /(n + 1) n→∞ n + 2 xn

n + 1
= lim |x|
n→∞ n + 2

1 + n1
= lim |x|
= |x|
n→∞ 1 + n2

For the series to converge, |x| < 1. Thus, the radius of convergence R = 1, and the
interval of convergence is −1 < x < 1. To test the inequality signs let x = −1, then
∞ ∞ ∞ ∞
X (−1)n (−1)n X (−1)2n X 1 X1
= = =
n=0
n+1 n=0
n+1 n=0
n + 1 n=1 n

This is a Harmonic series and thus divergent. So that x = −1 is not included in the
interval of convergence. Now let x = 1,
∞ ∞ ∞
X (−1)n (1)n X (−1)n X (−1)n−1
= =
n=0
n+1 n=0
n+1 n=1
n

By AST, the series converges (See Example 5.12). So that x = 1 is included in the
interval of convergence. Therefore the interval of convergence is −1 < x ≤ 1. 

Example 7.3:
Find the radius of convergence and the interval of convergence of the series

X (−1)n−1 xn
n=1
n3

26
Solution:
By Ratio Test,
n+1
(−1)n xn+1 /(n + 1)3 x n 3
lim = lim × n
n→∞ (−1)n−1 xn /n3 n→∞ (n + 1)3 x
 3
n
= lim |x|
n→∞ n+1
 3
1
= lim |x|
n→∞ 1 + n1
= |x|

The series converges if |x| < 1. Therefore, the radius of convergence R = 1. For find the
interval of convergence, we consider first at x = −1,
∞ ∞ ∞
X (−1)n−1 (−1)n X (−1)2n−1 X 1
= =−
n=1
n3 n=1
n3 n=1
n3

where 2n − 1 is an odd number for all integer n and thus (−1)2n−1 = −1. Since the last
series is a p–series with p = 3, thus the series converges. At x = 1,
∞ ∞ ∞
X (−1)n−1 (1)n X (−1)n−1 X (−1)n
= =−
n=1
n3 n=1
n3 n=1
n3

By AST where

i) {1/n3 } is a decreasing sequence, and


1
ii) limn→∞ n3
=0

the series converges. Therefore, the interval of convergence is −1 ≤ x ≤ 1. 

Example 7.4:
Find the radius of convergence and the interval of convergence of the series

X (−1)k x2k
k=0
(2k)!

27
Solution:
By Ratio Test,

ak+1 (−1)k+1 x2(k+1) /[2(k + 1)]!
lim = lim
k→∞ ak k→∞ x2k /(2k)!
2k+2
x (2k)!
= lim × 2k
k→∞ (2k + 2)! x
(2k)!
= lim |x|2
k→∞ (2k)!(2k + 1)(2k + 2)
1
= lim |x|2 = 0
k→∞ (2k + 1)(2k + 2)

Thus, the series converges for all x and the radius of convergence R = ∞ and interval
of convergence is (−∞, ∞). 

Example 7.5: P
Find the radius of convergence and the interval of convergence of the series ∞
n=1 an
where
(−2)n xn
an = √ 4
n
Solution:
By Ratio Test,

an+1 (−2)n+1 xn+1 / 4 n + 1
lim = lim √
n→∞ an n→∞ (−2)n xn / 4 n
r
n
= lim − 2x 4
n→∞ n + 1
s
1
= lim − 2x 4
1

n→∞ 1+ n
= | − 2x| = 2|x|

Thus, the series converges when 2|x| < 1 or |x| < 12 . The radius of convergence R = 12 .
At x = − 12 ,
(−2)n (− 12 )n X (−2)n (−2)−n X 1
X ∞ ∞ ∞
√ = 1 = 1
n=1
4
n n=1 n4 n=1 n
4

1
Since this is a p–series with p = 4
≤ 1, it is divergent at x = − 21 . When x = 12 ,

(−2)n ( 21 )n X (−2)n (2)−n X (−1)n


X∞ ∞ ∞
√ = √ = √
n=1
4
n n=1
4
n n=1
4
n

By AST,

28
 
1
i) √ is a decreasing sequence, and
4
n
1
ii) lim √ =0
n→∞ 4
n

the series converges by AST test. Therefore, the interval of convergence is − 12 < x ≤ 12 .


7.2 Power Series centered at x = a


If a is a constant, and if x is replaced with x − a, then the resulting series has the form

X
cn (x − a)n = c0 + c1 (x − a) + c2 (x − a)2 + · · · + cn (x − a)n + · · ·
n=0

which is known as the power series centered at x = a or about x = a.


E.g.,

X (x − 1)n (x − 1) (x − 1)2 (x − 1)n
i) =1+ + +···+ +···
n=0
n+1 2 3 n+1

X (−1)k (x + 3)k (x + 3)2 (x + 3)3
ii) = 1 − (x + 3) + − +···
k=0
k! 2! 3!

Theorem 5: P n
Given a power series ∞
n=0 cn (x − a) , it is exactly one of the following statements is
true:

i) The series converges only when x = a.


ii) The series converges for all x.
iii) There is a positive number R such that the series converges if |x − a| < R and
diverges if |x − a| > R.
iv) The series may converges or diverges at either of these values x = a−R or x = a+R.

If the series converges for |x − a| < R, then the value a is at the center between a − R
and a + R:
Example 7.6:
Find the radius of convergence and the interval of convergence for the series

X (x − 5)n
n=1
n2

29
R R

b
x
a
a−R a+R

Solution:
By Ratio Test,

(x − 5)n+1 /(n + 1)2 (x − 5)n+1 n2
lim = lim ×
n→∞ (x − 5)n /n2 n→∞ (n + 1)2 (x − 5)n
 2
n
= lim |x − 5|
n→∞ n + 1
 2
1
= lim |x − 5|
n→∞ 1 + 1
n
= |x − 5|
Thus, the series converges if |x − 5| < 1. The radius of convergence is R = 1. The
interval is
|x − 5| < 1 ⇒ −1 < x − 5 < 1
5−1 < x <5+1
4< x <6
To check the end points:
When x = 6,
∞ ∞ ∞
X (x − 5)n X (6 − 5)n X 1
= =
n=1
n2 n=1
n2 n=1
n2
This is a p–series with p = 2 > 1 and thus it is convergent.
When x = 4,
∞ ∞ ∞
X (x − 5)n X (4 − 5)n X (−1)n
2
= 2
=
n=1
n n=1
n n=1
n2
By AST,
 
1
i) is a decreasing sequence, and
n2
1
ii) lim = 0.
n→∞ n2

Thus, this series is convergent by AST. Therefore, the interval of convergence is 4 ≤ x ≤


6. 

30
Example 7.7:
Find the radius of convergence and the interval of convergence for the series

X k(x + 2)k
k=1
3k+1

Solution:
By Ratio Test,

(k + 1)(x + 2)k+1 /3k+2
lim
n→∞ k(x + 2)k /3k+1

(k + 1)(x + 2)k+1 k(x + 2)k
= lim ×
n→∞ 3k+2 3k+1

k + 1 x + 2
= lim ×
n→∞ k 3

x + 2
=

3

By Ratio Test, for the series to converge



x + 2
3 < 1, ∴ |x + 2| < 3

Therefore, the radius of divergence is R = 3. Now,

|x + 2| < 3 ⇒ −3 < x + 2 < 3


−5 < x <1

To check the endpoints:


At x = −5,
∞ ∞ ∞ ∞
X k(x + 2)k X k(−3)k X k(−1)k · 3k 1X
= = = (−1)k k
k=1
3k+1 k=1
3k · 3 k=1
3k · 3 3 k=1

This series is divergent.


At x = 1,
∞ ∞ ∞
X k(x + 2)k X k(3)k X k
= =
k=1
3k+1 k=1
3k · 3 k=1
3
This series is divergent. The interval of convergence is −5 < x < 1 or (−5, 1). 

∼ End ∼

31

You might also like